Integral: $\int_{0}^{1}\frac{\arctan^{2}\left(x\right)}{x}dx$

calculusdefinite integralsintegration

Context: While working on a contour integral for fun, I stumbled upon the following integral:

$$\int_{0}^{1}\frac{\arctan^{2}\left(x\right)}{x}dx.$$

I typed it into WolframAlpha and got that it equals

$$\frac{1}{8}(4\pi C – 7\zeta{(3)}),$$

where $C$ denotes Catalan's Constant and $\zeta{(3)}$ denotes Apery's Constant.

Attempt: Let's call the original integral $I$. At first, I tried IBP, then letting $x = \tan{(\theta)}$, then IBP again like this:

$$
\eqalign{
I &= -2\int_{0}^{1}\frac{\arctan\left(x\right)\ln\left(x\right)}{1+x^{2}}dx \cr
&= -2\int_{0}^{\frac{\pi}{4}}x\ln\left(\tan\left(x\right)\right)dx \cr
&= 2\int_{0}^{\frac{\pi}{4}}\frac{x^{2}}{\sin\left(2x\right)}dx.
}
$$

At that point, I decided I was using IBP an unnecessary amount of times and figured there has to be a nicer solution. I also tried differentiating with respect to a parameter $a$ and defining

$$J(a) = -2\int_{0}^{1}\frac{\arctan\left(x\right)\ln\left(ax\right)}{1+x^{2}}dx,$$

but I ended up circling back to where I started after doing a lot of grunt work. I also tried

$$ -2\int_{0}^{\frac{\pi}{4}}x\ln\left(\tan\left(x\right)\right)dx = -2\int_{0}^{\frac{\pi}{4}}x\ln\left(\sin\left(x\right)\right)dx+2\int_{0}^{\frac{\pi}{4}}x\ln\left(\cos\left(x\right)\right)dx$$

and using Taylor Series and complex definitions of $\sin{(x)}$ and $\cos{(x)}$, but I was getting a mess.

Question: Does anyone know a nice way of solving the given integral? If it's not a pretty solution, it's fine. Any hints and help are appreciated.

Best Answer

Continue with $$I= -2\int_{0}^{1}\frac{\arctan x\ln x}{1+x^{2}}\overset{x\to \frac1x}{dx}= \frac\pi2 \int_1^\infty \frac{\ln x}{1+x^2}dx -\int_0^\infty \frac{\arctan x\ln x}{1+x^2} dx$$ where $\int_1^\infty \frac{\ln x}{1+x^2}dx=G$ and \begin{align} \int_0^\infty \frac{\arctan x\ln x}{1+x^2}dx =& \int_0^\infty \int_0^1 \frac{x\ln x}{(1+x^2)(1+y^2x^2)} \overset{x\to \frac1{xy}}{dx}dy\\ = & \ \frac1{2}\int_0^1\int_0^\infty \frac{-x\ln y}{(1+x^2)(1+{y^2}x^2)} {dx}\ dy\\ =& \ \frac12\int_0^1\frac{\ln^2 y}{1-y^2}dy =\frac78\zeta(3) \end{align} Plug back into $I$ to ontain $$I= \frac\pi2G- \frac78\zeta(3)$$